Please find the surface area of this figure. Ps pls show your work on how you got your answer or explain.

Please Find The Surface Area Of This Figure. Ps Pls Show Your Work On How You Got Your Answer Or Explain.

Answers

Answer 1

Answer:

2(1/2)(11)(13) + 2(1/2)(10)(13.2) + 10(11)

= 143 + 132 + 110 = 385 square feet

B is the correct answer.


Related Questions

The following blueprint of a kitchen has dimensions of 7 inches by 7 inches. The island has been highlighted in red.


The island's actual dimensions are 3 1/2 feet by 1 3/4 feet. If the scale of the blueprint is 1 inch = 2 feet, what are the dimensions of the island on the blueprint?

Answers

The dimensions of the island on the blueprint are 14 inches by 3.5 inches.

We have,

The actual dimensions of the island are 3 1/2 feet by 1 3/4 feet.

We need to find the dimensions of the island on the blueprint, given that the scale of the blueprint is 1 inch = 2 feet.

To convert the actual dimensions to the dimensions on the blueprint, we need to use the scale factor of 1 inch = 2 feet.

We can set up a proportion to relate the actual dimensions to the dimensions on the blueprint:

Actual dimension/blueprint dimension = scale factor

Let x be the length of the island on the blueprint.

Then we can set up the following proportion:

3.5 feet / (1.75 feet)

= x inches / 7 inches

Simplifying,

2 = x / 7

Multiplying both sides by 7, we get:

x = 14 inches

The length of the island on the blueprint is 14 inches.

Similarly, we can find the width of the island on the blueprint:

1.75 feet / 3.5 feet

= y inches / 7 inches

Simplifying, we have:

0.5 = y / 7

Multiplying both sides by 7, we get:

y = 3.5 inches

The width of the island on the blueprint is 3.5 inches.

Thus,

The dimensions of the island on the blueprint are 14 inches by 3.5 inches.

Learn more about expressions here:

https://brainly.com/question/3118662

#SPJ1

Alice finds shirts on sale for $18.99.She buys twelve how much money does she spend?

Answers

Answer:

Well, if Alice buys 12 shirts which each cost $18.99 the equation would be 18.99 * 12 which = 227.88

Alice spent $227.88 on 12 shirts

Step-by-step explanation:

Answer:

Well, if Alice buys 12 shirts which each cost $18.99 the equation would be 18.99 * 12 which = 227.88

Step-by-step explanation:

Please help asap!!!!! I'm confused

Answers

The area of the parallelogram is 8.5 square miles. This is found by multiplying the length of one of the parallel sides, 2 miles, by the height, which is given as 4 1/4 miles.

To find the area of a parallelogram, we can multiply the length of one of its parallel sides by the length of its perpendicular height. Therefore, to find the area of this parallelogram, we need to determine its height.

We are given that one of the parallel sides has a length of 4 1/4 mi and the other has a length of 2 mi. We are also given that the length of the perpendicular on one of the parallel sides is 4 1/4 mi, which means that this is the height of the parallelogram.

So, the area of the parallelogram is

Area = base x height

Area = 2 mi x 4 1/4 mi

Area = 8 1/2 mi²

Therefore, the area of the parallelogram is 8 1/2 square miles or 8.5 square miles.

To know more about Area of parallelogram:

https://brainly.com/question/19187448

#SPJ1

geometry pls help fast 13 and 14

Answers

Answer:

13: (2,3) 14: A. Yes, 90, 180, 270 B. No C. No

Step-by-step explanation:

13. Right: -2+4, Down: 5-2

14. If it can be rotated and be similar, it has rotational.

Find the value of the following expression and round to the nearest integer:

Answers

The value of the Expression is 610, 919.

We have,

Expression : [tex]\sum_{n=0}^{61[/tex] 700 (1.07[tex])^{n+1[/tex]

The expression is a summation formula, representing the sum of a series of values.

Here r= 1.07 and n is number of terms.

So, [tex]\sum_{n=0}^{61[/tex] 700 (1.07[tex])^{n+1[/tex]

= 700 (1) +  [tex]\sum_{n=0}^{61[/tex] 700 (1.07[tex])^{n[/tex]

= 700 + 700 ([tex]r^{61[/tex] - 1)/ (r-1)

= 700 + 700 ([tex](1.07)^{61[/tex]-1)/ (0.07)

= 700+ 700 (871.7428)

= 610, 919

Learn more about Summation here:

https://brainly.com/question/29103782

#SPJ1

An insurance company offers an ordinary annuity that earns 6.5% interest compounded annually. A couple plans to make equal annual deposits into this account for 30 years and then make 20 equal annual withdrawals of €25,000, reducing the balance of the account to zero.

(i) Compute the value of the fund based on the withdrawals required. [5 marks]

(ii) Compute the amount of each deposit needed in order to maintain the fund. [5 marks]

(iii) Compute the total interest earned over the entire 50 years. [5 marks]​

Answers

Answer:

(i) To compute the value of the fund based on the withdrawals required, we can use the formula for the future value of an annuity due:

FV = P * ((1 + r)^n - 1) / r) * (1 + r)

where FV is the future value of the annuity, P is the annual payment, r is the interest rate per period, n is the total number of periods, and the extra (1 + r) factor is because the payments are made at the beginning of each period.

In this case, P = €25,000, r = 0.065, n = 20. We want to find the future value at the end of the 20-year period:

FV = 25000 * ((1 + 0.065)^20 - 1) / 0.065) * (1 + 0.065)

FV ≈ €743,704.96

Therefore, the value of the fund based on the withdrawals required is approximately €743,704.96.

(ii) To compute the amount of each deposit needed in order to maintain the fund, we can use the formula for the present value of an ordinary annuity:

PV = P * ((1 - (1 + r)^(-n)) / r)

where PV is the present value of the annuity, P is the annual payment, r is the interest rate per period, and n is the total number of periods.

In this case, PV = €743,704.96, r = 0.065, n = 20. We want to find the annual payment:

PV = P * ((1 - (1 + 0.065)^(-20)) / 0.065)

P ≈ €22,630.53

Therefore, the amount of each deposit needed in order to maintain the fund is approximately €22,630.53.

(iii) To compute the total interest earned over the entire 50 years, we can subtract the total deposits from the total withdrawals, and then subtract the initial balance. The total deposits are the annual deposit amount times the number of years (30), and the total withdrawals are the annual withdrawal amount times the number of years (20). The initial balance is the present value of the annuity that we calculated in part (ii).

Total deposits = €22,630.53 * 30 = €678,915.90

Total withdrawals = €25,000 * 20 = €500,000

Initial balance = €743,704.96

Total interest earned = Total withdrawals - Total deposits - Initial balance

Total interest earned = €500,000 - €678,915.90 - €743,704.96

Total interest earned ≈ -€922,620.86

Note that the negative sign indicates that the insurance company actually earned interest on this annuity, rather than the couple earning interest on their investment. This is because the withdrawals are greater than the deposits, and the interest rate earned by the insurance company is greater than the interest rate paid to the couple.

Step-by-step explanation:

A geography teacher assigns each student to write a report about one of the first 13 colonies. Students select the name of a colony by "blindly" drawing a colony's name from a bag. Once a colony has been drawn, it is not replaced.

What is the probability that the first student selects Pennsylvania and the second student selects Virginia? Round your answer to the nearest hundredth of a percent.

Answers

The probability that the first student selects Pennsylvania and the second student selects Virginia to the nearest hundredth of a percent is 0.64%.

Probability problem

The probability that the first student selects Pennsylvania is 1/13. Once Pennsylvania has been drawn, there are only 12 colonies left in the bag, so the probability that the second student selects Virginia is 1/12.

To find the probability that both events occur, we multiply the probabilities:

(1/13) x (1/12) = 1/156

To express this probability as a percentage, we multiply by 100:

(1/156) x 100 ≈ 0.64%

Therefore, the probability that the first student selects Pennsylvania and the second student selects Virginia is approximately 0.64%.

More on probability can be found here: https://brainly.com/question/30034780

#SPJ1

You can use the notation P(A), read “the probability of event B, given event A” to write a

A. Probability distribution
B. Frequency table
C. Conditional probability
D. Cumulative probability

Answers

You can use the notation P(A), read “the probability of event B, given event A” to write a conditional probability. The correct answer is C.

Conditional probability refers to the probability of one event occurring given that another event has already occurred. In this case, we are interested in the probability of event B occurring given that event A has already occurred, and we can represent this using the notation P(B|A), where '|' means 'given'.

For example, let's say we are interested in the probability of getting a head on a coin toss (event B), given that the coin was flipped and landed on heads (event A). We could represent this using the notation P(B|A). The value of P(B|A) would be 1, because if the coin already landed on heads, then the probability of getting a head on the next flip is certain.

Conditional probability is an important concept in probability theory and is often used in real-world applications, such as predicting the likelihood of a disease given certain symptoms, or the probability of an event occurring given certain conditions.

The correct answer is C.

To learn more about probability click on,

https://brainly.com/question/29259732

#SPJ1

construct a binomial whose gcf is 7a^3

Answers

The binomial whose greatest common factor is 7a³ is 14a⁴ + 35a³.

Given, the a³ is not common in A and C.

Now, in 14a⁴ + 35a³

14a⁴ = 2 x 7 x a³ x a

35a³ = 7 x 5 x a³

So, the common factors of 14 and 35 is 7.

Then, 14a⁴ + 35a³

= ( 2 x 7 x a³ x a) + (  5 x 7 x a³)

= 7a³ (2a + 5)

Learn more about greatest common factor here:

brainly.com/question/11221202

#SPJ1

In parallelogram ABCD, AE =3x +7and CE=x+25.
What is AC?
O 68
O 34
O 18
09
D
C
B

Answers

In parallelogram ABCD, AE = CE since opposite sides of a parallelogram are congruent. Therefore, 3x + 7 = x + 25. Solving for x, we get x = 9. Substituting x = 9 into either AE or CE gives us the length of AC. So, AC = 3x + 7 = 3(9) + 7 = 34. Therefore, the answer is B. 34.

Helppp I need the ending numbers ?

Answers

The solution is, the simplification of the equation is :

4x^2 - 4 = (2x -2 ) (2x+2)

Here, we have,

given that,

the expression is:

4x^2 - 4

now, we have to simplify this.

we know that ,

The a^2 - b^2 formula is also known as "the difference of squares formula". The a square minus b square is used to find the difference between the two squares without actually calculating the squares.

It is one of the algebraic identities.

It is used to factorize the binomials of squares.

The a^2 - b^2 formula is given as:

a^2 - b^2 = (a - b) (a + b).

so, we have,

4x^2 - 4

= (2x)^2 - 2^2

= (2x -2 ) (2x+2)

Hence, The solution is, the simplification of the equation is :

4x^2 - 4 = (2x -2 ) (2x+2)

To learn more on Equation:

brainly.com/question/10413253

#SPJ1

Find the angle between the pair of vectors to the nearest tenth of the degree

Answers

The value of angle between the two vectors is 86⁰.

What is the angle between the two vectors?

The value of angle between the two vectors is calculated as follows;

tan θ = vy/vx

where;

vy is the sum of the vertical directionvx is the sum of vectors in horizontal direction

( -8, 9), (-9, -6)

vy = (-6 - 9) = -15

vx = (-9 + 8) = -1

tan θ = ( -15 ) / ( -1 )

tan θ = 15

The value of θ is calculated  by taking arc tan of the fraction,;

θ = tan ⁻¹ ( 15 )

θ =  86⁰

Learn more about direction here: https://brainly.com/question/30318208

#SPJ1

Question 1 (1 point)
Write an inequality for the sentence.
The stadium held less than 25,000.
B
O a
O b
9ba580e611107c96c9efb866417dc160.webm 64 KB
s> 25,000
$≤25,000
Oc $<25,000

Answers

The inequality that represents the sentence "The stadium held less than 25,000 people" is given as follows:

c < 25,000.

What are the inequality symbols?

The four inequality symbols, along with their meaning on the number line and the coordinate plane, are presented as follows:

> x: the amount is greater than x -> the number is to the right of x with an open dot at the number line. -> points above the dashed horizontal line y = x on the coordinate plane.< x: the amount is less than x. -> the number is to the left of x with an open dot at the number line. -> points below the dashed horizontal line y = x on the coordinate plane.≥ x: the amount is at least x. -> the number is to the right of x with a closed dot at the number line. -> points above the solid vertical line y = x on the coordinate plane.≤ the amount is at most x. -> the number is to the left of x with a closed dot at the number line. -> points above the dashed vertical line y = x on the coordinate plane.

The amount is less than in this problem, hence the symbol is given as follows:

<.

As the amount is less than 25000, the inequality is given as follows:

c < 25,000.

More can be learned about inequalities at brainly.com/question/25275758

#SPJ1

Is this right. Don’t know the difference between the stratified and systematic random. Pic below

Answers

The difference between the stratified and systematic random sampling is explained below.

Systematic random sampling is a method where every Kth person of the population is chosen to be part of the sample, whereas stratified random sampling is a method where the population is first divided into subgroups, and then drawing a simple random sample from each subgroup.

Learn more about stratified and systematic random sampling click;

https://brainly.com/question/28963265

#SPJ1

how do you find the net of a rectangular prism

Answers

Answer:

The formula looks like this:

Surface Area = 2 l w + 2 l h + 2 h w ,

where SA = surface area, l = length, w = width, and h = height. In the rectangular prism net above, l = 8 inches, w = 5 inches, and h = 3 inches. Simply put these numbers into the formula and solve for surface area.

Brooklyn and Matthew both recently got a job and want to start a savings account to earn interest on money they save. Brooklyn's paycheck for was for $625 net pay-after taxes. the bank her parents use offer savings account compounded monthly with an interest rate of 2.5%. Matthews paycheck was also for $446 net pay. his parents know of a savings account that earns 4% compounded quarterly.
a. use the compounding interest rate formula to identify who will have more money after two years (assuming no additional deposits or withdrawals are made)
b. use the Desmos graphing calculator to graph Brooklyn's equation and Matthew’s equation on the same graph. identify the points of intersection and interpret the results.

WHAT MISTAKES DID THE STUDENTS MAKE??

Answers

After two years, Brooklyn will have more money in her savings account than Matthew.

Brooklyn's savings account grows at a slower rate than Matthew's initially but eventually catches up and surpasses Matthew's account due to the higher interest rate and monthly compounding.

a.

[tex]A = P(1 + r/n)^{nt}[/tex]

where A is the final amount, P is the initial principal (or net pay), r is the interest rate (as a decimal), n is the number of times the interest is compounded per year, and t is the number of years.

For Brooklyn saving account,

P = $625, r = 0.025, n = 12 (monthly compounding), and t = 2.

A = $625(1 + 0.025/12)^(12*2) = $686.71

For Matthew saving account,

P = $446, r = 0.04, n = 4 (quarterly compounding), and t = 2.

A = $446(1 + 0.04/4)^(4*2) = $506.84

b.

To graph Brooklyn's equation and Matthew's equation on the same graph, we can use the following equations:

Brooklyn's equation: A = 625(1 + 0.025/12)^(12t)

Matthew's equation: A = 446(1 + 0.04/4)^(4t)

Now,

The resulting graph will show the growth of each saving account over time.

The points of intersection on the graph represent the times when the two savings accounts have the same amount of money.

From the graph, we can see that the two accounts intersect at around 11 months and 23 months.

We can say that Brooklyn's savings account grows at a slower rate than Matthew's initially, but eventually catches up and surpasses Matthew's account due to the higher interest rate and monthly compounding.

Thus,

After two years, Brooklyn will have more money in her savings account than Matthew.

Brooklyn's savings account grows at a slower rate than Matthew's initially but eventually catches up and surpasses Matthew's account due to the higher interest rate and monthly compounding.

Learn more about saving accounts here:

https://brainly.com/question/13155407

#SPJ1

Which equation represents a direct variation?
Oy - 4x = 8
Oy + 2 = 7x
Oy - 3x = 0
O y = 5x - 2

Answers

The only equation that represents a direct variation is: y - 3x = 0

How to identify direct variation?

Direct Variation is defined as the relationship that exists between two variables in which one is a constant multiple of the other. For example, when one variable changes the other, then they are said to be in proportion. If b is directly proportional to a the equation is of the form b = ka (where k is a constant).

Making y the subject in each of the options gives:

A) y = 4x + 8

B) y = 7x - 2

C) y = 3x

D) y = 5x - 2

Looking at them, the only one where there is a relationship that exists between two variables in which one is a constant multiple of the other is option C

Read more about Direct variation at: https://brainly.com/question/6499629

#SPJ1

Can I get some help in this question?

What value of x satisfies the equation (7q^3x)^3=343q^36

Answers

4 is the value of the variable x.

What is algebraic Expression?

Any mathematical statement that includes numbers, variables, and an arithmetic operation between them is known as an expression or algebraic expression. In the phrase 4m + 5, for instance, the terms 4m and 5 are separated from the variable m by the arithmetic sign +.

We can simplify the left side of the equation by using the properties of exponents:

[tex](7q^{3x})^3 = 7^3 * (q^{3x})^3 = 343q^{9x}\\\\343q^{9x} = 343q^{36}\\\\q^{9x} = q^{36[/tex]

Now we can use the property that if [tex]a^b = a^c[/tex], then b = c. Therefore:

9x = 36

Dividing both sides by 9, we get:

x = 4

Therefore, the value of x that satisfies the equation is 4.

Learn more about algebraic Expression here:

https://brainly.com/question/953809

#SPJ1

What is the value of z?

Answers

Answer: z = 8

Step-by-step explanation:

The diagram shows us that 8z + 3z + 2 = 90

so we can say that: 11z = 88

therefore z = 8.

Note: diagrams can be misleading! this diagram technically shows us that 64 < 26!

A group consists of seven Democrats and eight Republicans. Four people are selected to attend a conference.
a. In how many ways can four people be selected from this group of fifteen?
b. In how many ways can four Republicans be selected from the eight Republicans?
c. Find the probability that the selected group will consist of all Republicans.
a. The number of ways to select four people from the group of fifteen is
b. The number of ways to select four Republicans from the group of eight Republicans is
c. The probability is

Answers

There 1365 ways to choose four people from the group of fifteen.

b. There are 70 ways to choose  four Republicans from the group of eight Republicans.

C. The probability is about  0.0513, or  5.13%.

What is the probability  about?

a.  To know the ways that four people can be selected from this group of fifteen is by:

nCr = n! / (r! x (n-r)!),

Where:

n  = total number of items

r = is the number of items to be selected,

!  = the factorial of a number.

Putting in the values into the the formula:

15C4 = 15! / (4! x (15-4)!)

(15-4)! = 11!

15C4 = 1365

B.  Since:

n = 8

r = 4

Putting in the values into the the formula:

8C4 = 8! / (4!  x (8-4)!)

(8-4)! = 4!

8C4 = 70

c. The  Probability = Number of ways to choose four Republicans / Number of ways to choose four people

Hence  Probability = 70 / 1365

                             =  0.0513

Therefore, the probability that the selected group will consist of all Republicans is about  0.0513, or  5.13%.

Learn more about probability  from

https://brainly.com/question/24756209

#SPJ1

A belt runs a pulley at 80 revolutions per minute. Find the angular velocity of the pulley in radians per second.

Answers

A belt runs a pulley at 80 revolutions per minute.

(a) To find the angular speed of the pulley in radians per second, we can use the formula

ω = 2πf

Where

ω = angular speed in radians per second

f = frequency in Hertz

We know that the pulley rotates at a rate of 80 revolutions per minute. To convert this to a frequency in Hertz, we can divide by 60 seconds per minute

f = 80 rev/min ÷ 60 min/s = 4/3 Hz

Now we can plug in this value for f into the formula

ω = 2πf = 2π(4/3) = 8π/3 rad/sec

Therefore, the angular speed of the pulley is 8π/3 radians per second.

(b) To find the linear speed of the belt in centimeters per second, we can use the formula

v = rω

Where

v = linear speed in centimeters per second

r = radius of the pulley in centimeters

ω = angular speed in radians per second

We know the radius of the pulley is given in centimeters. Let's assume it is r cm. We just found the angular speed to be 8π/3 radians per second. Now we can plug in these values into the formula

v = rω = r(8π/3) = (8π/3)r cm/s

Therefore, the linear speed of the belt is (8π/3)r centimeters per second.

The given question is incomplete and the complete question is ''A belt runs a pulley at 80 revolutions per minute. Find the angular velocity of the pulley in radians per second. (b)Find the linear speed of the belt in centimeters per second''.

To know more about revolutions here

https://brainly.com/question/29707118

#SPJ1

Give the rectangular coordinates for the point

Answers

The rectangular coordinates from the polar coordinates are: (-4√2, -4√2)

How to convert polar coordinates to rectangular coordinates?

The steps to  to convert polar coordinates to rectangular coordinates are:

Step 1: Find the x -coordinate for the rectangular coordinate form of the point by using the equation x = r cos(θ)

Step 2: Find the y -coordinate for the rectangular coordinate form of the point by using the equation y = rsin(θ)

Step 3: Write the rectangular coordinates as (x,y) using the results from steps 1 and 2.

We are given the polar coordinates as (8, 225°)

Thus:

x-coordinate of rectangular coordinate = 8 cos 225 = -4√2

y-coordinate of rectangular coordinate = 8 sin 225 = -4√2

Read more about rectangular coordinates at: https://brainly.com/question/29155884

#SPJ1

The distance between cities A and B on a map is 12.5 in. The distance from city B to city C, is 8.5 in, and the distance from C to A is 16.25 in. If the bearing
from A to B is N75°E, find the bearing from C to 4. Round to the nearest tenth of a degree.
16.25 in
12.5 in
8.5 in
The bearing from city C to city 4 is approximately (Choose one) (Choose one)

Answers

The bearing of C to A is 239⁰.

What is the bearing of C to A?

The bearing of C to A is calculated by finding the value of angle C using cosine rule since we know the value of all the sides of the triangle.

AB² = AC² + CB² - 2(AC)(CB) cosC

12.5² = 16.25² + 8.5² - 2(16.25 x 8.5) x cos C

156.25 = 336.3125 - 276.25cosC

276.25cosC = 180.06

cosC = 180.06/276.25

cos (C) = 0.6518

C = cos⁻(0.6518)

C = 49.3⁰

The value of angle A is calculated as follows;

Sin A/CB = Sin C/AB

sin A/8.5 = sin 49.3/12.5

sin A = 8.5 [sin 49.3/12.5]

sin A = 0.5157

A = sin⁻¹ (0.5157)

A = 31⁰

The bearing of C to A is calculated as;

= 270⁰ - 31⁰

= 239⁰

Learn more about bearing here: https://brainly.com/question/28782815

#SPJ1

Answer all questions please

Answers

The value of f(1) is 3.

An estimate of the value of f(-1) is -0.2.

The values of x for which f(x) = 1 are: x = (0, 3).

The value of x such that f(x) = 0 is x = -0.5.

The domain of f is {-2, 4}.

The range of f is {-1, 3}.

The interval over which f is increasing is {-2, 1}.

What is a domain?

In Mathematics and Geometry, a domain is the set of all real numbers for which a particular function is defined.

Furthermore, the vertical extent of any graph of a function represents all range values and they are always read and written from smaller to larger numerical values, and from the bottom of the graph to the top.

By critically observing the graph of the function shown in the image attached above, we can reasonably and logically deduce the following domain and range:

Domain = {-2, 4}.

Range = {-1, 3}.

In conclusion, we can logically deduce that this function is increasing over the [-2, 1] and decreasing over the interval [1, 4].

Read more on domain here: brainly.com/question/17440903

#SPJ1

Forty people were asked their favorite kind of pizza. Thirty percent of the people surveyed chose sausage. How many people preferred sausage?

Answers

To find out how many people preferred sausage, you can start by multiplying the percentage who chose sausage, which is 30%, by the total number of people surveyed, which is 40.

So, 30% of 40 is:

0.30 x 40 = 12

Therefore, 12 people preferred sausage.

Answer: 12 people

Step-by-step explanation:

0.30 x 40 = 12

math hw for tonight
help solve this problem! Thank you!
ap cal bc

Answers

Answer:

first option

Step-by-step explanation:

differentiate using the power rule

[tex]\frac{d}{dx}[/tex] (a[tex]x^{n}[/tex] ) = na[tex]x^{n-1}[/tex]

then

[tex]\frac{dy}{dx}[/tex] = [tex]\frac{dy}{dt}[/tex] × [tex]\frac{dt}{dx}[/tex] = [tex]\frac{\frac{dy}{dt} }{\frac{dx}{dt} }[/tex]

y = t² + 4t

[tex]\frac{dy}{dt}[/tex] = 2t + 4

x = t - 3

[tex]\frac{dx}{dt}[/tex] = 1

then

[tex]\frac{dy}{dx}[/tex] = [tex]\frac{2t+4}{1}[/tex] = 2t + 4

Answer:

2t + 4

Step-by-step explanation:

A parametric equation is one where x and y are defined separately in terms of a third variable (often the parameter t).

To find dy/dx from parametric equations, differentiate each equation with respect to the parameter t, then use the chain rule:

[tex]\boxed{\dfrac{\text{d}y}{\text{d}x}=\dfrac{\text{d}y}{\text{d}t} \times \dfrac{\text{d}t}{\text{d}x}}[/tex]

Differentiate the two parametric equations with respect to t:

[tex]x=t-3 \implies \dfrac{\text{d}x}{\text{d}t}=1[/tex]

[tex]y=t^2+4t \implies \dfrac{\text{d}y}{\text{d}t}=2t+4[/tex]

Use the chain rule to combine them:

[tex]\begin{aligned}\implies \dfrac{\text{d}y}{\text{d}x}&=\dfrac{\text{d}y}{\text{d}t} \times \dfrac{\text{d}t}{\text{d}x}\\\\&=(2t+4) \times \dfrac{1}{1}\\\\&=2t+4\end{aligned}[/tex]

Therefore:

[tex]\boxed{\dfrac{\text{d}y}{\text{d}x}=2t+4}[/tex]

A curve, described by x2 + y2 + 6y = 0, has a point A at (−3, −3) on the curve.

Part A: What are the polar coordinates of A? Give an exact answer.

Part B: What is the polar form of the equation? What type of polar curve is this?

Part C: What is the directed distance when theta equals 4 pi over 3 question mark Give an exact answer.

Answers

a) The polar coordinates of point A are (√(18), π/4).

b) The curve is a circle centered at the origin with radius 6.

c) The directed distance is the value of r, which is 6 √(3).

To find the polar coordinates of point A on the curve, we need to convert the point from Cartesian to polar coordinates. The conversion formula is:

r = √(x² + y²)

θ = arctan(y/x)

Using the values of point A, we have:

r = √((-3)² + (-3)²) = √(18)

θ = arctan((-3)/(-3)) = arctan(1) = π/4

To find the polar form of the equation x² + y² + 6y = 0, we need to convert it from Cartesian to polar coordinates. The conversion formulae are:

x = r cos(θ)

y = r sin(θ)

Using these formulae, we can rewrite the equation as:

r² cos²(θ) + r² sin²(θ) + 6r sin(θ) = 0

Simplifying this equation, we get:

r = -6 sin(θ) / (1 - cos²(θ))

To find the directed distance when θ equals 4 π over 3, we need to substitute this value of θ into the polar equation we found in Part B. Doing so, we get:

r = -6 sin(4 π/3) / (1 - cos²(4 π/3))

r = -6(-√(3)/2) / (1 - (-1/2)²)

r = 6 √(3)

To know more about polar coordinates here

https://brainly.com/question/31422978

#SPJ1

Need help pls and thank u! ​

Answers

Answer:

[tex]\sqrt[]{72}[/tex]

6.3141414141414...

[tex]\sqrt[4]{64}[/tex]

8.121 121 112 111...

Step-by-step explanation:

irrational numbers are real numbers that cannot be expressed as a ratio of integers.

example of irrational number [tex]\sqrt[]{2}[/tex]

example of rational number 2, 3, -4, etc.

Margarita fue a la tienda y compro una cartera y unos jeans por un
total de RD$3,250. Sabiendo que las cartera excede al jeans en
RD$970, ¿Cuántos pago margarita por cada artículo?
Cartera = RDS
Jeans = RD$
Escribir las respuestas numéricas y sin comas.
OK

Answers

The solution is , price of jeans = RD$ 1140 and, price of  handbag =

RD$ 2110.

Here, we have,

given that,

Margarita went to the store and bought a bag and some jeans for a total of RD$3,250.

Knowing that the handbag exceeds the jeans by RD$970,

now, we have to find that, how many do she pay for each article.

let, price of jeans = RD$ x

so, price of  handbag = RD$ (x +970)

ATQ, we get,

RD$ x + RD$ (x +970) = RD$3,250

or, RD$ 2x + 970 = RD$3,250

or, RD$ 2x = RD$ 2280

or, x = RD$ 1140

Hence, price of jeans = RD$ 1140 and, price of  handbag = RD$ 2110.

To earn more on addition click:

brainly.com/question/29560851

#SPJ1

Need help. This please

Answers

The domain of the quadratic function in this problem is given as follows:

All real values.

How to obtain the domain of the function?

The domain of a function is the set of all the possible input values that can be assumed by the function.

On the graph, the domain of the function is given by the values of x of the function.

A quadratic function has no restrictions on the domain, hence it is defined by all the real values.

More can be learned about the domain of a function at https://brainly.com/question/10687170

#SPJ1

Other Questions
If 3.28g of a gas occupies a volume of 6.22 liters at a pressure of 845mmHg and a temperature of 378k A) how many moles of gas exist in the container?B) what is the molar mass of the gas?SHOW YOUR WORK!!!! What diagnosis ofPneumothorax (Chest Pain DDX) Determine over what interval(s) (if any) the Mean Value Theorem applies. (Enter your answer using interval notation. If an answer does not exist, enter DNE.)y=x225 How many triangles are there? the resource conservation and recovery act of 1976 a. requires the epa to set standards for hazardous-waste management. b. requires all firms that handle more than 100 kilograms of hazardous waste per month to have a permit stating how such wastes are to be managed. c. provides guidelines and financial aid to establish state waste management programs. d. all of these answers e. none of these answers Suppose a city contains 200,000 registered voters. Of these, 120,000 (60%) support aparticular ballot proposition to legalize recreational marijuana at the state level.Suppose 200 of the voters are randomly selected to be polled, and all of them actuallyrespond to the poll and report their beliefs truthfully.(a) The process of choosing 200 voters at random and counting the total number whosupport the ballot proposition is like drawing 200 times without replacement from abox. Describe or draw this box.(b) If we drew 200 times with replacement from this box, there is a 95% chance that thesum of draws (number of voters contacted who support the ballot proposition) wouldbe between ________ and _______. Expressed as a percentage of the 200 people, this isbetween ________% and _______%. Lela's teacher showed the class the image above. She explained that the image is a small crystal of salt. Lela's teacher gave the class the following information:Some molecules bond to other molecules in a pattern. These groups of molecules are called crystals because they have a crystalline structure. They are made of molecules that join to other molecules that are the same.Salt molecules are made of sodium and chlorine, two elements (atoms) that join together to make a salt molecule. The sodium is smaller than the chlorine.Which of the following is TRUE? Quien era luciano en mi planta de naranja lima Laptop computers are made with batteries, but they must also be plugged into outlets to charge the batteries. Which is true regarding laptops? Corrosion occurs when there is a _____ differential between two components of a systemA) currentB) voltageC) supplyD) pHE) carbon assume that a ball of charged particles has a uniformly distributed negative charge density except for a narrow radial tunnel through its center, from the surface on one side to the surface on the opposite side. also assume that we can position a proton anywhere along the tunnel or outside the ball. let fr be the magnitude of the electrostatic force on the proton when it is located at the ball's surface, at radius r. as a multiple of r, how far from the surface is there a point where the force magnitude is 0.31fr if we move the proton in the following ways? Katrina is building a wall around her backyard using bricks that are 18 inches long. The wall needs to be 36 feet, or 432 inches, long. Top management usually performs all of the following tasks EXCEPT:A. establish long-range plansB. review new product developmentC. develop strategic plansD. direct workers on the assembly lineE. formulate basic policies of operation Sheridan Snowboarding Company, a public company, purchased equipment on January 10, 2017, for $660,000. At that time, management estimated that the equipment would have a useful life of 10 years and a residual value of $50,000. Sheridan uses the straight-line method of depreciation and has a December 31 year end.Sheridan tested the equipment for impairment on December 31, 2021, after recording the annual depreciation expense. It was determined that the equipments recoverable amount was $289,000, and that the total estimated useful life would be eight years instead of 10, with a residual value of $10,000 instead of $50,000.What will appear on Sheridan's 2021 balance sheet with regard to this equipment?Sheridan Snowboarding CompanyBalance Sheet (Partial)Assets$Add/Less: 2. Assuming no further impairments or recoveries, calculate the annual depreciation expense for the years 2022 to 2024. (Round depreciation rate to 2 decimal places, e.g. 15.75 and final answers to 0 decimal places, e.g. 5,275.)Depreciation Expense2022 $2023 $2024 $ the sticky-price theory of the short-run aggregate supply curve says that if the price level rises by 5% while firms were expecting it to rise by 2%, then some firms with high menu costs will have question 10 options: higher than desired prices, which leads to an increase in the aggregate quantity of goods and services supplied. higher than desired prices, which leads to a decrease in the aggregate quantity of goods and services supplied. lower than desired prices, which leads to an increase in the aggregate quantity of goods and services supplied. lower than desired prices, which leads to a decrease in the aggregate quantity of goods and services supplied. When searching a building, rescuers should use more than one search method and ideally blend all three methods, which are: The starting point that is studied in relation to the degree of change following a promotional campaign is called a:A) post-hoc analysisB) marginal analysisC) benchmark measureD) standardized measure Suppose log 2 = r, log, 3= s, and log 5 = t. Which algebraic expression represents log 75? Which of the following are TRUE about the Sprint Retrospective? (Choose two.) Which of the following are TRUE about the Sprint Retrospective? (Choose two.)A. It is three hours for a one month SprintB. It occurs before the Sprint ReviewC. It is an opportunity to inspect the people, relationships, process, and tools in the last SprintD. It is the only time improvements are made during a Sprint Which of these lines best explains the relationship between the past and future in the poem?